1
$\begingroup$

I am not a mathematics researcher but I am concerned that this question, posed with slightly different wording on math.stackexchange, may be too esoteric for that forum since it concerns the details of a proof that I don't think is standard in the graduate level curriculum. At least, nobody has answered it so far.

Let $G$ be a group with presentation $<x_1,x_2...,x_m|R>$ and let $G'=G \ast<y_0>$.

Now define $y_i=y_{0}x_i$. Notice that $G'=<y_0,y_1,...y_m|R'>$ for appropriately defined $R'$ and that $<y_i>$ is free on $y_i$ for all $i$.

Consider the group

$$H = <y_0,y_1,...y_m,t_1,t_2,...t_m,|R',t_iy_it^{-1}_i=y_{i}^2>$$

Either take my word for it that $G'$ embeds (isomorphically) into $H$, or notice that, since $<y_i>$ is free on $y_i$ for all $i$, $H$ is an iterated HNN extension on $G'$ and therefore embeds $G'$ by $m$ applications of Britton's Lemma.

Now let $w$ be a nonidentity element in G. Define $H^{'}$ as $H$ subject to the relation $(w^{-1}y_{0}^{-1}wy_0)t_i(w^{-1}y_{0}^{-1}wy_0)^{-1} = t_{i}^2$ for each $t_i$.

I can show that $w^{-1}y_{0}^{-1}wy_0$ is infinite order since it is cyclically reduced word not containing the identity and length $\geq 2$. $t_i$ is also infinite order in $H$ since it is the pivot letter in an HNN extension. So it would seem that I'm not introducing any torsion for the elements $y_0,t_i$.

I also don't think I'm introducing any torsion for elements of $G$ since it does not reference them at all.

In particular it seems like $G'$ embeds into $H'$. I have no idea how to prove this, but I have done pages of calculations in this group to try and disprove this.

+++++++++++++++++

Is this true? Here's why I think no, in spite of my best efforts to (dis)prove it:

In the proof of the Adian-Rabin theorem, $G$ is the free product of some group with unsolvable word problem $U$ with a group $G_{-}$, which has the property that any group it embeds into does not satisfy some property $M$. (A $G_{-}$ with such a property is given by hypothesis).

The proof constructs $H$, and then uses $H$ in the construction of another group, call it $K$. $K$ has the property that it embeds $G_{-}$ iff $w\neq1$, and is trivial otherwise. Proving that $K$ depends on $w$ in this way is the crux of the theorem. However, I think that my group $H'$ also has the desired property w.r.t. $w$, which would mean that the proof could be simplified somewhat.*

All proofs of Adian-Rabin that I have seen go as far as constructing $K$, they don't stop at $H$ or construct $H'$, so there must be something about my construction that makes it not necessarily embed $G_{-}$. I cannot figure out what that could be, for reasons outlined above.

*NB: In $H'$, $w=1$ implies $w^{-1}y_{0}^{-1}wy_0=1$ implies the $t_i$'s and thus the $y_i$'s all equal 1. The proof that $K$ is trivial goes exactly like this too, except it involves the extra relations used to construct $K$.

New contributor
Perry Bleiberg is a new contributor to this site. Take care in asking for clarification, commenting, and answering. Check out our Code of Conduct.
$\endgroup$
1

0

Your Answer

Perry Bleiberg is a new contributor. Be nice, and check out our Code of Conduct.

By clicking “Post Your Answer”, you agree to our terms of service and acknowledge that you have read and understand our privacy policy and code of conduct.